Answer B
So my thought process for this question eliminated Answer B almost immediately because B says: ...
KiaBrodersen on January 9 at 02:56PM
  • June 2017 LSAT
  • SEC3
  • Q13
1
Reply
Why eliminate fuel rods?
I'm a little confused about the question explanation and how it rules out fuel rods. (I might be ...
devon on June 12, 2022
  • June 2017 LSAT
  • SEC3
  • Q24
2
Replies
Why not A (Circular Logic)
Ok, I read the Key Takeaway that in an Errors in Reasoning question that features conditional log...
Karen-Norris on March 5, 2022
  • June 2017 LSAT
  • SEC3
  • Q23
1
Reply
Why is A correct? Why is D incorrect?
Thanks
Ryan-Mahabir on February 8, 2022
  • June 2017 LSAT
  • SEC3
  • Q14
7
Replies
Explain
Can you please explain why C is right and why the other answer choices are not? Thank you.
Irene-Vera on February 7, 2022
  • June 2017 LSAT
  • SEC3
  • Q16
3
Replies
Question 21
isn't the argument giving an example of the general claim that evolution does not always optimize...
Zahra on February 6, 2022
  • June 2017 LSAT
  • SEC3
  • Q21
5
Replies
a vs. d
Why is A wrong?
schicago on August 17, 2020
  • June 2017 LSAT
  • SEC3
  • Q17
1
Reply
Why is the answer not D?
I don't understand how to eliminate D to get to the right asnwer.
ruchitaj on August 6, 2020
  • June 2017 LSAT
  • SEC3
  • Q22
2
Replies
Help
Can someone help me with this plz
alymathieu on July 21, 2020
  • June 2017 LSAT
  • SEC3
  • Q19
3
Replies
Question 4 Why is E right?
So when I saw this question the first time I chose D. After seeing that I got it wrong, I tried a...
evelynakinyemi on July 6, 2020
  • June 2017 LSAT
  • SEC3
  • Q4
1
Reply
Can't figure out the diagramming for this argument
Can you please diagram the conditional statements for this argument? I seem to be arriving at con...
Golnar-Roughani on June 14, 2020
  • June 2017 LSAT
  • SEC3
  • Q24
3
Replies
June 2017 SEC 3 Q12
For questions like this one, do we presuppose that the answers are valid and look for the ones th...
kens on May 19, 2020
  • June 2017 LSAT
  • SEC3
  • Q12
1
Reply
Question explanation
Could you break down this question? Is the conclusion the first or the last sentence? I couldn't ...
wills on April 24, 2020
  • June 2017 LSAT
  • SEC3
  • Q17
2
Replies
What type of question is this?
Hi! Always thrown by these questions.
TimB on March 10, 2020
  • June 2017 LSAT
  • SEC3
  • Q8
1
Reply
Why is B incorrect?
Thanks in advance
devinjax14 on February 23, 2020
  • June 2017 LSAT
  • SEC3
  • Q12
1
Reply
Please Help
I will appreciate a breakdown of this, I do not understand it.Thanks
toyalli on January 31, 2020
  • June 2017 LSAT
  • SEC3
  • Q18
2
Replies
B
Why is B wrong?
tomgbean on January 1, 2020
  • June 2017 LSAT
  • SEC3
  • Q15
1
Reply
A
How do different criteria equate to flawed methodology?
tomgbean on January 1, 2020
  • June 2017 LSAT
  • SEC3
  • Q11
1
Reply
C
Why is C wrong? is it similar to A?
tomgbean on January 1, 2020
  • June 2017 LSAT
  • SEC3
  • Q9
1
Reply
why not A?
I chose A because the stimulus talks about piano players, and then jumps to a conclusion that inc...
tori06koto on December 24, 2019
  • June 2017 LSAT
  • SEC3
  • Q9
2
Replies